Partial derivative of function w.r.t. the percent change of the variable

Click For Summary
The discussion focuses on rewriting the partial derivative of a function with respect to the percent change of a variable. The user attempts to express the relationship between the change in the function and the change in the variable using logarithmic differentiation. They clarify that the expression for the percent change in the function relative to the percent change in the variable can be derived from the relationship between the function and its logarithm. The user is confused about how to express the change in the function when the variable increases by 1%, contrasting it with a 1 unit increase. Ultimately, they seek to confirm that the correct expression for this relationship is ∂f(x,y)/∂log(x) = x∂f(x,y)/∂x.
Usuiisu
Messages
3
Reaction score
0

Homework Statement



Rewrite this in terms of f, f, ∂f/∂x, and x:
∂f(x,y)/∂(%Δx) = ∂f(x,y)/∂(d log(x) )

Homework Equations



∂(%Δf(x,y))/∂(%Δx) = ∂logf(x,y)/∂log(x)= ∂f(x,y)/∂x*x/f(x,y).

∂f(x,y)/∂log(x)=x∂f(x,y)/∂x

The Attempt at a Solution



I found that (%Δx) can be written as the differential of log(x):
∂f(x,y)/∂(%Δx) = ∂f(x,y)/∂(d log(x) ).

But the partial of derivative of differential of the variable throws me off. It seems like the valid problems that people may have in real world: how much the level of function changes if you increase 1% of the variable, but I can't find anything on it.
 
Physics news on Phys.org
I made a mistake in the relevant questions for one of the equations:

%Δf(x,y)/%Δx = ∂logf(x,y)/∂log(x)= ∂f(x,y)/∂x*x/f(x,y)
should be corrent.
 
I am now confused. The quantity that I want to get is that
how much f(x,y) changes if you increase x by 1%.
If you say, "how much f(x,y) changes if you increase x by 1 unit", then
it should be ∂f(x,y)/∂x.
So, what I want is: Δf(x,y)/%Δx.
Can you write this as, ∂f(x,y)/∂log(x)? Then, the answer is x∂f(x,y)/∂x.
 
Question: A clock's minute hand has length 4 and its hour hand has length 3. What is the distance between the tips at the moment when it is increasing most rapidly?(Putnam Exam Question) Answer: Making assumption that both the hands moves at constant angular velocities, the answer is ## \sqrt{7} .## But don't you think this assumption is somewhat doubtful and wrong?

Similar threads

  • · Replies 2 ·
Replies
2
Views
1K
  • · Replies 4 ·
Replies
4
Views
1K
Replies
26
Views
4K
  • · Replies 8 ·
Replies
8
Views
2K
  • · Replies 6 ·
Replies
6
Views
2K
  • · Replies 7 ·
Replies
7
Views
2K
Replies
2
Views
2K
  • · Replies 28 ·
Replies
28
Views
3K
  • · Replies 4 ·
Replies
4
Views
1K
  • · Replies 3 ·
Replies
3
Views
2K